EMT 230 Cardiology Flashcards

1
Q

Your acute coronary syndrome patient has a 12-lead indicating he is having an anterolateral wall MI. The patient denies any shortness of breath. His pulse oximeter reading is 93% on room air. Which of the following is true regarding oxygen administration?
A) Oxygen is not necessary as the patient denies any shortness of breath
B) Oxygen should be administered at 12-15 lpm to increase pulse ox to 100%
C) Oxygen should be provided at 4 lpm to avoid constricting the coronary arteries
D) Oxygen should be provided at 6 lpm via nasal canula to decrease anxiety yet increase pulse ox reading

A

C) Oxygen should be provided at 4 lpm to avoid constricting the coronary arteries

How well did you know this?
1
Not at all
2
3
4
5
Perfectly
2
Q

Which of the following statements is TRUE regarding the cardiovascular system?
A) The right side of the heart pumps at high pressure to the lungs
B) The largest muscle mass is found within the walls of the left ventricle
C) Capillaries are responsible for returning blood to the heart for oxygenation
D) The pulmonary vein is responsible for delivering oxygen rich blood to the lungs

A

B) The largest muscle mass is found within the walls of the left ventricle

How well did you know this?
1
Not at all
2
3
4
5
Perfectly
3
Q
The ability of cardiac cells to depolarize without any outside course is known as the property of:
A) Contractility 
B) Conductivity 
C) Automaticity 
D) Rhythmicity
A

C) Automaticity

How well did you know this?
1
Not at all
2
3
4
5
Perfectly
4
Q
An electrical impulse exits the SA node and is next conducted to which of the following:
A) Purkinjie fibers
B) Bundle of HIS
C) Internodal pathways
D) Atrioventricular junction
A

D) Atrioventricular junction

How well did you know this?
1
Not at all
2
3
4
5
Perfectly
5
Q
The middle layer of vessel responsible for giving blood vessels their ability to constrict and dilate is referred to as which of the following:
A) Tunica intima 
B) Tunica media
C) Tunica adventitia 
D) Tunica arteria
A

B) Tunica media

How well did you know this?
1
Not at all
2
3
4
5
Perfectly
6
Q
You are called to a patient with difficulty breathing. You arrive to find the patient has an extensive history of CHF and you assess the patient for neck vein distension. This is best done at a degree angle.
A) 15
B) 25
C) 45
D) 90
A

C) 45

How well did you know this?
1
Not at all
2
3
4
5
Perfectly
7
Q
The process of depolarization is dependent on which of the following?
A) Osmosis 
B) Sodium filtration 
C) the levels of potassium 
D) the sodium-potassium pump
A

C) the levels of potassium

How well did you know this?
1
Not at all
2
3
4
5
Perfectly
8
Q
Which of the following vascular emergencies would NOT require an emergency, lights and sirens transport?
A) Aortic aneurysm
B) Arterial occlusion 
C) Deep vein thrombosis
D) Pulmonary embolism
A

C) Deep vein thrombosis

How well did you know this?
1
Not at all
2
3
4
5
Perfectly
9
Q
In the event of a coronary artery blockage, how could the muscle of the heart still receive blood?
A) Anastomoses
B) Aorta
C) Vasoconstriction 
D) Coronary arteries
A

A) Anastomoses

How well did you know this?
1
Not at all
2
3
4
5
Perfectly
10
Q
Which of the following is a branch of the right coronary artery?
A) Posterior ascending
B) Left coronary artery 
C) Circumflex 
D) Acute marginal artery
A

D) Acute marginal artery

How well did you know this?
1
Not at all
2
3
4
5
Perfectly
11
Q
Which of the following is your highest priority for a patient who is unresponsive, hypotensive, and has a bradycardia dysrhythmia?
A) Identify the cause
B) Administer 
C) Initiate transcutaneous pacing 
D) Secure the airway with an ET tube
A

C) Initiate transcutaneous pacing

How well did you know this?
1
Not at all
2
3
4
5
Perfectly
12
Q
You have positioned a patient complaining of chest pain on the EMS stretcher. The patient will MOST likely prefer which position for transport?
A) Tri-pod
B) Semi-fowlers
C) Lying supine
D) Trendelenburg
A

B) Semi-fowlers

How well did you know this?
1
Not at all
2
3
4
5
Perfectly
13
Q
What is the time period represented on the EKG, when the ventricular cells are repolarizing and are preparing for another nerve impulse?
A) P wave 
B) PR interval 
C) ST segment 
D) T wave
A

D) T wave

How well did you know this?
1
Not at all
2
3
4
5
Perfectly
14
Q
The inability of the heart to meet the metabolic needs of the body would be the definition for which of the following?
A) Cardiogenic shock 
B) Hypertensive crisis 
C) Cardiac tamponade 
D) Myocardial infarction
A

A) Cardiogenic shock

How well did you know this?
1
Not at all
2
3
4
5
Perfectly
15
Q

For which of the following rhythms is transcutaneous pacing usually CONTRAINDICATED?
A) Third degree heart block
B) Pulseless electrical activity
C) Second degree block type II
D) Bradycardia with hemodynamic instability

A

B) Pulseless electrical activity

How well did you know this?
1
Not at all
2
3
4
5
Perfectly
16
Q
Which of the following is NOT a component of the autonomic nervous system?
A) The spinal cord 
B) The sympathetic nervous system 
C) The parasympathetic nervous system 
D) The cardiac plexus
A

A) The spinal cord

How well did you know this?
1
Not at all
2
3
4
5
Perfectly
17
Q

Inotropy is defined as which of the following?
A) Speed of electrical conduction heart rate
B) reperfusion of the heart
C) Force of cardiac contraction
D) Heart muscle size

A

C) Force of cardiac contraction

How well did you know this?
1
Not at all
2
3
4
5
Perfectly
18
Q
You have recently intubated a patient in respiratory arrest. The patient now has a heart rate of 30. There is a normal PR interval and the QRS complex measure 0.10 seconds. Which of the following is MOST likely the cause of the dysrhythmia in this patient?
A) hypovolemia
B) Vagal stimulation 
C) Excessive caffeine 
D) Myocardial infarction
A

B) Vagal stimulation

How well did you know this?
1
Not at all
2
3
4
5
Perfectly
19
Q
Which of the following is typically NOT used in the standard approach to evaluation ECG rhythm strips?
A) Analyzing the rate
B) Analyzing the rhythm 
C) Analyzing the PR interval 
D) Analyzing the ST segment
A

D) Analyzing the ST segment

How well did you know this?
1
Not at all
2
3
4
5
Perfectly
20
Q

Which of the following is the correct definition of Starling’s law?
A) The more myocardial muscle stretched the greater its force of contraction
B) the greater systemic vessel resistance the greater the force of contraction
C) As cardiac output decreases force of myocardial contractions decrease
D) The greater the parasympathetic stimulation, the greater the heart is stretched

A

A) The more myocardial muscle stretched the greater its force of contraction

How well did you know this?
1
Not at all
2
3
4
5
Perfectly
21
Q
The intrinsic rate of the SA node is:
A) 15-40 per minute
B) 40-60 per minute
C) 60-80 per minute
D) 60-100 per minute
A

D) 60-100 per minute

How well did you know this?
1
Not at all
2
3
4
5
Perfectly
22
Q
Which of the following ions is MOST responsible for ensuring cardiac muscle contraction?
A) Magnesium 
B) Chloride
C) Calcium 
D) Sodium
A

C) Calcium

How well did you know this?
1
Not at all
2
3
4
5
Perfectly
23
Q
An ECG can measure all of the following except:
A) Heart rate
B) Rhythm 
C) Cardiac output
D) Electrical voltage
A

C) Cardiac output

How well did you know this?
1
Not at all
2
3
4
5
Perfectly
24
Q
Which of the following ECG waveforms represent s atrial depolarization?
A) The QRS complex
B) The T wave
C) The P wave
D) The ST segment
A

C) The P wave

How well did you know this?
1
Not at all
2
3
4
5
Perfectly
25
Q

You have administered aspirin, nitro, and an analgesic to a patient with a previous complaint of chest pain who is now pain free and refusing transport. Which of the following would be the BEST therapeutic communication technique for the patient to reconsider his decision?
A) He is within the window of opportunity for thrombolytics
B) Once he has received analgesic, he cannot refuse treatment
C) Protocol does not allow you to discontinue care once an IV is established
D) He could go into arrest any minute and he should be at the hospital if this happens

A

A) He is within the window of opportunity for thrombolytics

How well did you know this?
1
Not at all
2
3
4
5
Perfectly
26
Q
Which of the following cardiac conduction abnormalities occurs frequently and can be detected with single-lead monitoring?
A) AV blocks
B) Hemiblocks
C) Bundle branch blocks
D) Chamber enlargements
A

A) AV blocks

How well did you know this?
1
Not at all
2
3
4
5
Perfectly
27
Q
Which of these is considered a major risk factor for cardiovascular disease?
A) Liver disease
B) Overall mental health
C) Individual response to stress
D) Diabetes mellitus
A

D) Diabetes mellitus

How well did you know this?
1
Not at all
2
3
4
5
Perfectly
28
Q
You are transcutaneously pacing who had a complete heart block prior to your intervention. the patient remains unresponsive and apneic and you are continuing ventilations. Which of the following is your MOST important intervention?
A) Obtaining a 12 lead ECG
B) Verifying electrical capture
C) verifying mechanical capture
D) setting the heart rate appropriately
A

C) verifying mechanical capture

How well did you know this?
1
Not at all
2
3
4
5
Perfectly
29
Q
leads I, II, III are also referred to as which of the following?
A) Precordial leads
B) Augmented leads
C) Unipolar leads
D) Bipolar leads
A

D) Bipolar leads

How well did you know this?
1
Not at all
2
3
4
5
Perfectly
30
Q
On ECG graph paper, an upward deflection represents:
A) Artifact
B) An isoelectric impulse
C) A positive electrical impulse
D) A negative electrical impulse
A

C) A positive electrical impulse

How well did you know this?
1
Not at all
2
3
4
5
Perfectly
31
Q
When attaching the leads of an ECG monitor, a lead II tracing requires the positive electrode to be placed on the:
A) Left arm 
B) Left leg
C) Right arm 
D) Right leg
A

B) Left leg

How well did you know this?
1
Not at all
2
3
4
5
Perfectly
32
Q
Sinus tachycardia is MOST commonly caused by:
A) Increased sympathetic tone 
B) Increased parasympathetic tone
C) Cocaine overdose
D) Cardiogenic shock
A

A) Increased sympathetic tone

How well did you know this?
1
Not at all
2
3
4
5
Perfectly
33
Q
What may be a lethal treatment for a patient with a ventricular escape rhythm?
A) Atropine
B) Dopamine
C) Transcutaneous pacing 
D) Lidocaine
A

D) Lidocaine

How well did you know this?
1
Not at all
2
3
4
5
Perfectly
34
Q
Which of the following ECG findings would be the MOST expected in a patient experiencing angina?
A) normal 12 lead
B) ST segment elevation 
C) ST segment depression 
D) Widened QRS complex
A

C) ST segment depression

How well did you know this?
1
Not at all
2
3
4
5
Perfectly
35
Q
Which of the following terms is defined as the passage of an electrical current through the heart during a specified part of the cardiac cycle to terminate certain kinds of dysrhytmias?
A) Defibrillation 
B) Transcutaneous 
C) Synchronized cardioversion 
D) Unsynchronized cardioversion
A

C) Synchronized cardioversion

How well did you know this?
1
Not at all
2
3
4
5
Perfectly
36
Q
Of what branch if the circumflex artery?
A) Left coronary artery 
B) Left anterior descending artery 
C) Right coronary artery 
D) Marginal artery
A

A) Left coronary artery

How well did you know this?
1
Not at all
2
3
4
5
Perfectly
37
Q
You have a patient with a normal heart rate and rhythm who is having a hypertensive emergency. Which of the following medications would address the hypertension while at the same time having little or no effect on the the heart?
A) beta blocker 
B) ACE inhibitor 
C) Calcium channel blocker
D) Combined alpha/beta antagonist
A

B) ACE inhibitor

How well did you know this?
1
Not at all
2
3
4
5
Perfectly
38
Q

Which of the following is NOT true of the U waves?
A) The usually appear positively deflected
B) They may be associated with electrolyte imbalance
C) They follow T waves
D) They are always present in patients

A

D) They are always present in patients

How well did you know this?
1
Not at all
2
3
4
5
Perfectly
39
Q

Which of the following signs and symptoms would most closely be associated with a patient in cardiogenic shock?
A) Tachycardia, warm skin and altered level of consciousness
B) Hypotension, cyanosis, bradycardia and a strong bounding pulse
C) Hypertension, altered level of consciousness chest pain and JVD
D) Hypotension, Hypoxia, and tachycardia, and pedal and/or pulmonary edema

A

D) Hypotension, Hypoxia, and tachycardia, and pedal and/or pulmonary edema

How well did you know this?
1
Not at all
2
3
4
5
Perfectly
40
Q
You are utilizing the R-R interval method to calculate a heart rate on an ECG. You have counted 30 small boxes. What is the heart rate of this ECG?
A) 36 
B) 42 
C) 50 
D) 72
A

C) 50

How well did you know this?
1
Not at all
2
3
4
5
Perfectly
41
Q

You are called to assess and treat a 75-year-old male with a primary complaint of weakness, nausea, mild chest pain, and syncope upon rising over the past two hours. The patient has a pulse that corresponds with the monitored rhythm shown, respirations are 24, blood pressure is 80/40, and the skin is cool and clammy. Which of the following BEST describes the appropriate field impression and treatment for this patient? (3rd degree heart block)
A) New onset second degree type II, fluids
B) New onset complete heart block; pacing
C) Increased parasympathetic tone, atropine
D) New onset myocardial infarction; morphine, oxygen nitro and aspirin

A

B) New onset complete heart block; pacing

How well did you know this?
1
Not at all
2
3
4
5
Perfectly
42
Q
Which of the following methods can be used for calculating heart rate and utilizing the ECG rhythm?
A)six second method
B) R-R method
C) Triplicate method
D) all of the above
A

D) all of the above

How well did you know this?
1
Not at all
2
3
4
5
Perfectly
43
Q
Which of the following is an appropriate rule for determining an idioventricular rhythm>
A) The pacemaker site is at the SA node
B) The P waves are inverted 
C) The rate is greater than 60 
D) The QRS is > 0.12
A

D) The QRS is > 0.12

How well did you know this?
1
Not at all
2
3
4
5
Perfectly
44
Q
Which of the following ECG abnormalities would be MOST indicative of myocardial ischemia?
A) Pathological Q waves
B) Physiological Q waves
C) An elevated ST segment 
D) A depressed ST segment
A

D) A depressed ST segment

How well did you know this?
1
Not at all
2
3
4
5
Perfectly
45
Q
Which of the following is NOT a typical cause of PEA?
A) Hypovolemia
B) Hypothermia 
C) Hypertension 
D) Hyperkalemia
A

C) Hypertension

46
Q

While treating a patient who was in cardiac arrest, you notice ventricular tachycardia on the monitor, your treatment will be:
A) Synchronized cardioversion, this patient is stable
B) Defibrillation, the patient is unstable
C) Lidocaine, there is time for the medication to work
D) Epinephrine, this will convert the rhythm

A

B) Defibrillation, the patient is unstable

47
Q
You are on scene of a patient experiencing chest pressure and SOB for approximately 3 hours without relief. The patient has a history of hypertension and angina pectoris. The patient complains of an upset stomach and has pale, diaphoretic skin. The assessment findings in this patient reveal which of the following?
A) Empohysema
B) Myocardial infarction 
C) Appendicitis
D) Pneumonia
A

B) Myocardial infarction

48
Q

Which of the following is the MOST important reason why we confirm breath sounds after placing an advanced airway?
A) An improperly placed advanced airway can be fatal
B) To recognize if we have a right main stem intubation
C) To demonstrate to the family competency in the skill
D) Assessing breath sounds is part of the standard of care

A

A) An improperly placed advanced airway can be fatal

49
Q

Who does the paramedic have legal responsibilities to?
A) The patient and the employer
B) The patient, the employer and medical director
C) The patient, medical director and the public
D) The patient, the employer, the medical director and the public

A

D) The patient, the employer, the medical director and the public

50
Q
Second Degree Type II heart block occurs when the impulse is not conducted through what portion of the cardiac electrical system?
A) SA node 
B) AV node 
C) Purkinjie fibers
D) Bundle branches
A

D) Bundle branches

51
Q

You are evaluating a patient with a possible cardiac problem. How would you expect this patient to describe his pain if he was experiencing a myocardial infarction?
A) “It hurts on the right side of my chest when I cough”
B) “It seems to hurt more when I take a deep breath in.”
C) “I feel pressure right under my breastbrone.”
D) “It is a sharp pain that seems to come and go.”

A

C) “I feel pressure right under my breastbrone.”

52
Q

A terminal cancer patient has a signed valid DNR. The patient’s family requests transport to the hospital and comfort measures such as IV fluid administration and pain medication. How should the paramedic handle this situation?
A) Provide no care but transport the patient; the DNR takes precedence
B) Provides BLS care only
C) Provide appropriate care; DNR orders do not interfere with non-resuscitative care
D) Provide no care and do not transport the patient; the DNR takes precedence

A

C) Provide appropriate care; DNR orders do not interfere with non-resuscitative care

53
Q

Your 68-year-old male patient complains of shortness of breath and generalized weakness and he states that he suddenly awoke in severe respiratory distress. He reports no allergies and he regularly takes Lanoxin and Furosemide. The patient is now sitting on the side of his bed upright with his arms supporting the rest of his body. He has a productive cough and weak peripheral pulses. There is no peripheral edema or jugular venous distention. When you place your stethoscope on the patient’s chest, what do you expect to hear?
A) A friction rub due to inflammation of the pleural space
B) Stridor and wheezing, evident of upper airway obstruction
C) Rales, rhonchi, and wheezing, consistent with severe pulmonary edema
D) Clear lung sounds bilaterally as the patient is displaying signs of right sided heart failure

A

C) Rales, rhonchi, and wheezing, consistent with severe pulmonary edema

54
Q
Which of the following would be the greatest hazard associated with the usage of an artificial external pacemaker?
A) Discomfort for the patient 
B) Damage to the myocardium 
C) Failure to verify mechanical capture 
D) The presence of a runaway pacemaker
A

C) Failure to verify mechanical capture

55
Q
The right atrium receives blood from the system circulation and the\_\_\_?
A) Left ventricle 
B) Coronary veins 
C) Pulmonary arteries 
D) Pulmonary veins
A

B) Coronary veins

56
Q
What is the name of the time period represented in a normal Lead II EKG for depolarization to travel through the atria and the AV node?
A) P wave 
B) PR interval 
C) R wave 
D) ST segment
A

B) PR interval

57
Q

You are assessing a patient you suspect is suffering from a cardiogenic shock. Which of the following assessment techniques would best help you differentiate whether this is a left sided issue?
A) Pedal edema
B) Pulmonary edema
C) ST elevation in II, II, aVF and V4
D) A positive response to a fluid challenge

A

B) Pulmonary edema

58
Q
You have a patient with a rapidly responding atriall fibrillation. Which of the following medication would provide this patient with the GREATEST benefit?
A) Beta agonist
B) Sodium channel blocker
C) Calcium channel blocker
D) Potassium channel blocker
A

C) Calcium channel blocker

59
Q

Which progressive event typically causes the cardiogenic shock patient to lose the ability to maintain vital organ perfusion?
A) Myocardial death
B) Failure of Starling’s law mechanism
C) Failure of preload, afterload and contractility
D) Failure of respiration secondary to massive pulmonary edema

A

C) Failure of preload, afterload and contractility

60
Q

You are called to assess a patient complaining of chest pain(9 out of 10) who shows evidence of a left ventricular infarct on a 12-lead. Which of the following would be your greatest anticipated problem?
A) The development of hypotension
B) The development of pulmonary edema
C) The progression into a life-threatening rhythm
D) The potential the patient will throw a pulmonary embolism

A

C) The progression into a life-threatening rhythm

61
Q
You have a patient who is unresponsive, has spontaneous respirations, and has a carotid pulse of 40 and absent radial and femoral pulses. Your ECG analysis is a Second Degree Type II and the 12 lead shows ST elevation in leads V1 and V2. Which of the following is your MOST appropriate intervention?
A) Atropine 
B) Dopamine Drip 
C) Epinephrine drip 
D) Transcutaneous pacing
A

D) Transcutaneous pacing

62
Q
Which of the following would you MOST likely see on the ECG monitor for a patient with elevated potassium levels?
A) Inverted P waves
B) Elevated ST segment 
C) Tall, peaked T waves
D) Abnormally narrow QRS complex
A

C) Tall, peaked T waves

63
Q
According to national standards, a patient experiencing a myocardial infarction(STEMI) should receive reperfusion therapy within how many hours after onset in order for the benefits to outweigh the risks?
A) 6 hours
B) 12 hours
C) 18 hours
D) 24 hours
A

B) 12 hours

64
Q
A patient experiencing acute congestive heart failure with associated pulmonary edema may display all of the following adventitious lung sounds EXCEPT:
A) Crackles
B) Stridor
C) Wheezes
D) Rhonchi
A

B) Stridor

65
Q
You have recently intubated a patient in respiratory arrest. The patient now has a carotid heart rate of 30. There is a normal PR interval and the QRS complex measures 0.10 seconds. These findings MOST likely indicate which of the following?
A) Pulseless electrical activity
B) First-degree heart block 
C) Sinus bradycardia
D) Sinus arrest
A

C) Sinus bradycardia

66
Q

You have a patient who is responsive to painful stimuli, is breathing at 24 times per minute, has a carotid pulse of 40, absent radial and femoral pulses, and is pale, cool, and clammy. You are unable to obtain a blood pressure, your ECG analysis is of a Second Degree Type II, and the 12-Lead shows ST elevation in Leads V1 and V2. You have decided to pace this patient and as a result, he wakes up and is in a tremendous amount of pain from the pacemaker. He also indicates that he is allergic to all of the pain relieving medications you have on board. What would be your MOST appropriate course of action based on this information?
A) Discontinue the pacemaker
B) Administer an analgesic and epi sub-Q
C) Start an Epi drop and then once you see it is working. DC the pacer
D) Explain the situation to the patient and recommend continued pacing

A

D) Explain the situation to the patient and recommend continued pacing

67
Q

Which of the following accuratley describes the relationship between stroke volume, heart rate, and cardiac output?
A)Stroke volume / heart rate = cardiac output
B) Stroke volume x heart rate = cardiac output
C) Cardiac output x heart rate = stroke volume
D) Cardiac output x stroke volume = heart rate

A

B) Stroke volume x heart rate = cardiac output

68
Q
You are assessing a patient complaining of chest pain with a left arm blood pressure of 100/70 and a right arm blood pressure of 140/90. Which of the following would MOST likely be the cause of the unequal pressures?
A) Pericardial tamponade
B) Left sided heart failure
C) Abdominal aortic aneurysm 
D) Peripheral artery disease
A

C) Abdominal aortic aneurysm

69
Q

Which of the following would indicate possible pacemaker failure?
A) A pacemaker spike preceding the T wave
B) A QRS complex duration of 0.20 seconds
C) Pacemaker spikes not following by a QRS complex
D) Occasional QRS complexes without pacer spikes

A

C) Pacemaker spikes not following by a QRS complex

70
Q
The MOST common cause of death for patients experiencing acute myocardial infarction is:
A) Ventricular aneurysm 
B) Lethal dysrhythmias
C) Pulmonary embolus 
D) Heart failure
A

B) Lethal dysrhythmias

71
Q

Which of the following would BEST describe why we provide oxygen to a patient having a myocardial infarction ?
A) To reduce myocardial oxygen demand
B) To vasodilate cardiac blood vessels
C) To provide additional oxygen to the brain
D) To oxygenate the collateral cardiac circulation

A

D) To oxygenate the collateral cardiac circulation

72
Q
Which of the following methods would be the BEST to use for calculating heart rate utilizing ECG rhythm when the ECG is irregular?
A) R-R interval 
B) Triplicate method
C) six second method
D) heart rate calculator ruler
A

C) six second method

73
Q
Which of the following events would have the greatest potential to produce a myocardial infarction?
A) Tension pneumothorax
B) Myocardial contusion 
C) Esophageal varices
D) Major arterial bleed
A

B) Myocardial contusion

74
Q
Which of the following is NOT a laboratory assessment for determining if a patient is suffering from acute myocardial infarction?
A) CK
B) CK-MB
C) Troponin
D) KCL
A

D) KCL

75
Q
According to the AHA, which of these is the typical initial heart rate setting for transcutaneous pacing of the bradycardiac patient?
A) 30 per minute
B) 50 per minute
C) 60 per minute
D) 100 per minute
A

C) 60 per minute

76
Q
Which of the following represents the appropriate maintenance regimen for Amiodarone in a post resuscitation phase?
A) 0.5 mg/min
B) 150 mg/min
C) 1.0 to 4.0 mg/min
D) 2.0 to 10.0 mg/min
A

A) 0.5 mg/min

77
Q
During a cardiac arrest you administer epinephrine, atropine, sodium bicarbonate and Amiodarone. The patient now has spontaneous respirations and pulse. Patients BP is 110/P. Which of the following is MOST likely to be administered as a maintenance medication?
A) Amiodarone
B) Epinephrine
C) Lidocaine
D) Sodium Bicarbonate
A

A) Amiodarone

78
Q
You are called to the scene of a 47 yeard old male with a current chief complaint of chest pain. The patient informs you he has never been sick a day in his life prior to this. Which of the following will provide the GREATEST information?
A) Allergies
B) OPQRST
C) Medications
D) DCAP-BTLS
A

B) OPQRST

79
Q
Which of the following is the MOST influential in determining stroke volume?
A) Venous return 
B) Afterload
C) Arterial pressure
D) Myocardial size
A

A) Venous return

80
Q

You are called to assess and treat a 62-year-old male with general weakness, pedal edema and a blood pressure of 90/50. His 12-lead indicates ST elevation in Leads II, III, AVF and V4R. His oxygen saturation is 90%. Which of the following would be the MOST appropriate field assessment and treatment for this patient?
A) Left sided MI; oxygen, nitro and aspirin
B) Unstable angina; aspirin and position of comfort
C) Right sided MI; fluid challenge aspirin and oxygen
D) Right sided heart failure; furosemide oxygen and elevation of feet

A

A) Left sided MI; oxygen, nitro and aspirin

81
Q

Your patient has atrial fibrillation, a rapid ventricular response and a history of Wolf-Parkinson-White (WPW) syndrome. Your partner suggests the administration of 6 mg of adenosine and you suggest this is incorrect. Which of the following BEST explains why?
A) The dosage of adenosine in this case should be 12 mg vs 6 mg because of WPW
B) The adenosine will not have any impact on this patient because of history of WPW
C) The adenosine will only temporarily slow the atrial fibrillation but will not stop the fast response
D) The adenosine will block the AV node, allowing the patient to go into ventricular fibrillation

A

D) The adenosine will block the AV node, allowing the patient to go into ventricular fibrillation

82
Q

Which of the following should be your highest priority in assessing and treating a conscious patient you suspect having a myocardial infarction?
A) The administration of aspirin
B) The collection of a complete history
C) The assessment of equal radial pulses
D) The identification of patient risk factors

A

A) The administration of aspirin

83
Q

For which of the following patients would a three-lead ECG analysis be the MOST useful?
A) An unresponsive trauma patient
B) A patient complaining of chest pain
C) A patient complaining of a racing heart
D) An unresponsive patient with rigor mortis

A

C) A patient complaining of a racing heart

84
Q
Which of the following interventions would be the MOST appropriate selection to increase the blood pressure of a 47-year-old hypotensive, bradycardic (HR-55) patient with ST elevation in Leads V1 and V2 and chest pain so as to receive nitro and/or morphine?
A) Atropine 2-10 mcg/min
B) Fluid challenge 1000-2000ml
C) Dopamine drip 2-10 mcg/kg/min
D) Epi drip 2-10 mcg/kg/min
A

C) Dopamine drip 2-10 mcg/kg/min

85
Q

Which of the following would BEST describe why we provide morphine to a patient having a myocardial infarction?
A) To reduce myocardial oxygen demand
B) To vasodilate cardiac blood vessels
C) To provide additional oxygen to the brain
D) To oxygenate the collateral cardiac circulation

A

A) To reduce myocardial oxygen demand

86
Q
Which of the following would clearly indicate that a myocardial infarction is coming?
A) Chest pain radiating to the jaw
B) ST elevation in 2 contiguous leads
C) ST depression in 2 contiguous leads
D) A decrease in cardiac enzymes levels
A

B) ST elevation in 2 contiguous leads

87
Q

Why should the administration of CPAP be considered in patients with pulmonary edema?
A) CPAP can deliver very high concentrations of oxygen
B) CPAP can keep the lungs inflated and the upper airway open
C) CPAP pressure help bring the left and right heart into homeostasis
D) CPAP provides ventilation pressure within the alveoli that may reduce fluid entering the alveoli

A

D) CPAP provides ventilation pressure within the alveoli that may reduce fluid entering the alveoli

88
Q
Which of the following cardiac drug may cause a paradoxical reaction when administered too slowly, potentially increasing bradycardia?
A) Lidocaine
B) Epinephrine 
C) Amiodarone
D) Atropine
A

D) Atropine

89
Q
Your patient has a history of SVT and left sided myocardial infarction. The patient now has a narrow complex tachycardia at a rate of 180, pulmonary edema, a blood pressure of 70/40, and is unresponsive. Which of the following should be your initial treatment?
A) Lasix 
B) Dopamine
C) Dobutamine
D) Synchronized cardioversion
A

D) Synchronized cardioversion

90
Q

Which of the following patients would be the BEST candidate for thrombolytic therapy?
A) A 63 yeard old male complaining of chest and jaw pain for 2-3 days
B) A 74 year old female with septal wall myocardial infarction and previous history of stroke
C) A 49 year old male suffering from a lateral wall infarction diagnosed within the last 2 hours
D) A 55 yeard old with an anterior wall infarction with abdominal pain following an MVC

A

C) A 49 year old male suffering from a lateral wall infarction diagnosed withi

91
Q
What property of dopamine causes an increased heart rate?
A) Alpha agonist 
B) Alpha antagonist
C) Beta agonist
D) Beta antagonist
A

C) Beta agonist

92
Q
The hallmark of atrial fibrillation is:
A) 1:3 conduction through the AV node
B) Multifocal PVCs
C) P waves following the QRS complex
D) An irregularly irregular rhythm
A

D) An irregularly irregular rhythm

93
Q

Which of the following would be a LATE sign of left sided heart failure?
A) Acute onset of dypsnea
B) The presence of pink frothy sputum
C) The presence of jugular vein distension
D) A heart rate of 120

A

B) The presence of pink frothy sputum

94
Q

You have just decided to terminate resuscitative measures at the residence of a patient. Your new basic EMT partner has asked why you did not transport the patient to the emergency department where additional measures would be available. Which of the following would be your BEST response?
A) You cannot run a code effectively in an ambulance
B) Transport would have been more costly for the family
C) Transport would have raised false hopes for the family
D) Our care is better than the hospitals’s for this type of call

A

C) Transport would have raised false hopes for the family

95
Q
If a patient had a decrease in cardiac output secondary to myocardial damage, how would the body initially a!empt to compensate to maintain an adequate blood pressure?
A) Decrease heart rate
B) increase fluid retention 
C) Decrease stroke volume
D) Increase vascular resistance
A

D) Increase vascular resistance

96
Q

A specific problem that atherosclerosis can cause is:
A) Arterial rupture
B) Ventricular rupture
C) Coronary blood vessel rupture
D) Interference with dilation/constriction of arteries

A

D) Interference with dilation/constriction of arteries

97
Q

Which of these BEST explains why we push hard, push fast and minimize interruptions when performing chest compression?
A) To circulate oxygen
B) To circulate the blood
C) To stop the heart from fibrillating
D) To maintain an adequate perfusion pressure

A

D) To maintain an adequate perfusion pressure

98
Q
Inotropy is defined as which of the following?
A) Speed of electrical conduction 
B) Heart rate 
C) Force of cardiac contraction 
D) Heart muscle size
A

C) Force of cardiac contraction

99
Q

You have just contacted medical direction for permission to terminate a resuscitation a!empt. Which of the following information is the physician LEAST likely to consider when deciding to terminate the resuscitation?
A) Therapy rendered
B) Medical condition of the patient
C) Quality of life evaluation performed by EMS
D) Resistance or uncertainty on behalf of the family

A

C) Quality of life evaluation performed by EMS

100
Q
You are performing CPR on a medical patient in cardiac arrest who was feeling fine 30 minutes ago and you are now unable to obtain any pulses. The patient has a history of myocardial infarction and your monitor shows a sinus tachycardia at 130. The patient has just gone into asystole despite all your efforts and you have ruled out and treated all possible causes. More than 20 minutes has elapsed, which of the following would be the MOST appropriate intervention?
A) Terminate the code
B) Administer epinephrine 
C) Administer sodium bicarbonate 
D) Establish transcutaneous pacing
A

A) Terminate the code

101
Q

Which of the following BEST describes a tachycardia of ventricular origin versus one of supraventricular origin?
A) A wide QRS and the absence of P waves
B) A wide QRS and a rate greater than 100
C) A wide QRS and positively deflected AVR
D) A wide QRS and negatively deflected AVR

A

B) A wide QRS and a rate greater than 100

102
Q
The heart sound heard by closure of the semilunar valves is which of the following?
A) S1
B) S2
C) S3 
D) S4
A

B) S2

103
Q
Which of the following choices below best represents the reason we give aspirin to patients we suspect of having a heart attack?
A) Platelet aggregation inhibitor 
B) Thrombolytic agent 
C) Blood thinner 
D) Analgesic
A

A) Platelet aggregation inhibitor

104
Q

You respond to a community recreation center for a 51-year-old male with severe chest pain. He was about to start his squash game when he suddenly developed this pain, which is now radiating to his lower jaw, neck, and back. Further assessment reveals tachycardia, nausea, and cool, clammy skin. You understand that the cause of these symptoms is due to:
A) Muscle strain from swinging the squash racquet too hard
B) Indigestion from heavy lunch the patient consumed a couple of hours ago
C) Stimulation of parasympathetic nervous system to compensate for decreased cardiac output
D) Imbalance between coronary artery supply and myocardial demand creating an oxygen deficiency to the heart muscle

A

D) Imbalance between coronary artery supply and myocardial demand creating an oxygen deficiency to the heart muscle

105
Q

Which of the following is the primary reason we give nitro to patients we think are having a heart attack?
A) For the analgesic properties
B) For its Beta 1 and Beta 2 properties
C) For its potent vasodilator properties
D) For its blood pressure lowering properties

A

C) For its potent vasodilator properties

106
Q
What is the group of nerves the innervates the atria and ventricles known as?
A) Brachial plexus
B) Aortic plexus
C) Cardiac plexus
D) Carotid plexus
A

C) Cardiac plexus

107
Q
Atropine may be given by all of the following routes EXCEPT:
A) IV push 
B) ET tube 
C) IO
D) SubQ
A

D) SubQ

108
Q

Which of the following defines afterload?
A) The resistance against which the heart must pump
B) The volume filling the ventricles during systole
C) The volume filling the ventricles during diastole
D) The amount of blood pumped in one minute

A

A) The resistance against which the heart must pump

109
Q
In which of the following situations would the act of performing an ECG analysis be of the LEAST benefit?
A) Chest pain 
B) Cardiac arrest
C) Difficulty breathing 
D) Multisystem trauma
A

D) Multisystem trauma

110
Q

In which of the following situations would you LEAST likely initiate resuscitative measures?
A) a 14 year old male just pulled from a cold swimming pool
B) a 64 year old female found unresponsive and cool to the touch
C) An 18 year old patient found unresponsive with dependent lividity
D) An 18 month old female who is unresponsive pulseless and dehydrated

A

C) An 18 year old patient found unresponsive with dependent lividity

111
Q
A 10-year-old female presents with cardiac arrest. The parents state that the child has had vomiting and diarrhea for 3 days. She has no other medical history and there are no signs of trauma. The most likely explanation for the cardiac arrest is:
A) Septic shock 
B) Hypoglycemic shock 
C) Electrolyte imbalance
D) Sudden infant death syndrome
A

C) Electrolyte imbalance